math1998

New Member
ارسال ها
336
لایک ها
224
امتیاز
0
پاسخ : ماراتن جبر (سطح پیشرفته)

f تابعی روی اعداد حقیقی است.

[img/]
 

AHZolfaghari

Well-Known Member
ارسال ها
935
لایک ها
1,654
امتیاز
93
پاسخ : ماراتن جبر (سطح پیشرفته)

f تابعی روی اعداد حقیقی است.

[img/]
[/SIZE][/SIZE][/QUOTE]
اول اثبات می کنیم تابع f پوشا و یک به یک هستش که این راحته . x رو ثابت فرض میکنیم و y رو از منفی بی نهایت یا مثبت بی نهایت تغییر میدیم و بدین ترتیب روی اعداد حقیقی پوشا هستش
فرض می کنیم [IMG]http://latex.codecogs.com/gif.latex?f%28y_%7B1%7D%29%20%3D%20f%28y_%7B2%7D%29 و x رو ثابت میذاریم از اینجا بدست میاد که
پس یک به یکه.
چون پوشا هست پس یه k وجود داره که
حالا جای x میذاریم k بدست میاد که

حالا جای x بذارید صفر بدست میاد که
پس (f(0 برابر با صفر هستش و چون یک به یکه k=0
حالا به جای x بذارید (f(x طرف چپ همون میشه اما طرف راست میشه
پس تابع f تابع همانی یا تابع
هستش
 

Dadgarnia

New Member
ارسال ها
1,350
لایک ها
1,127
امتیاز
0
پاسخ : ماراتن جبر (سطح پیشرفته)

ببخشید راه حل من برای سوال قبل غلط بود؟
 
آخرین ویرایش توسط مدیر

mohy1376

Well-Known Member
ارسال ها
418
لایک ها
311
امتیاز
63
پاسخ : ماراتن جبر (سطح پیشرفته)

ببخشید راه حل من برای سوال قبل غلط بود؟

باید پوشا بودن و یک به یک بودن رو ثابت میکردی بعد ادامه میدادی!
 

Dadgarnia

New Member
ارسال ها
1,350
لایک ها
1,127
امتیاز
0
پاسخ : ماراتن جبر (سطح پیشرفته)

باید پوشا بودن و یک به یک بودن رو ثابت میکردی بعد ادامه میدادی!
میشه بگید کجای اثباتم از یک به یک بودن و پوشا بودن استفاده کردم؟
اگه منظورتون اول اثباته که واضح است چنین aای وجود داره فقط کافیه به جای y قرار بدید
 

mohy1376

Well-Known Member
ارسال ها
418
لایک ها
311
امتیاز
63
پاسخ : ماراتن جبر (سطح پیشرفته)

میشه بگید کجای اثباتم از یک به یک بودن و پوشا بودن استفاده کردم؟
اگه منظورتون اول اثباته که واضح است چنین aای وجود داره فقط کافیه به جای y قرار بدید
آره خوب ، به نظرم مشکلی نداره اگه اولش اینو میگفتی.:39:
 

AHZolfaghari

Well-Known Member
ارسال ها
935
لایک ها
1,654
امتیاز
93
پاسخ : ماراتن جبر (سطح پیشرفته)

یه مشکلی وجود داره تو اثبات من و جناب
Dadgarnia و اون اینه که الان اثبات شده که ما برای هر x داریم که (f(x یا خود x هستش یا قرینش اما نمیدونیم f همه اعداد به یک فرم هست
بدین منظور فرض میکنیم برای اعداد a,b داریم :

که این تناقضه چرا که f هر عدد یا خودشه یا قرینش اما
نه قرینه
هستش نه خودشه پس نتیجه میگیریم f همه اعداد به یک فرم هست.:3::3::3::3:
 

Dadgarnia

New Member
ارسال ها
1,350
لایک ها
1,127
امتیاز
0
پاسخ : ماراتن جبر (سطح پیشرفته)

یه مشکلی وجود داره تو اثبات من و جناب
Dadgarnia و اون اینه که الان اثبات شده که ما برای هر x داریم که (f(x یا خود x هستش یا قرینش اما نمیدونیم f همه اعداد به یک فرم هست
بدین منظور فرض میکنیم برای اعداد a,b داریم :

که این تناقضه چرا که f هر عدد یا خودشه یا قرینش اما
نه قرینه
هستش نه خودشه پس نتیجه میگیریم f همه اعداد به یک فرم هست.:3::3::3::3:
ببخشيد من الان نه فهميدم شما چي رو مي خواين اثبات كنين نه نحوه ي اثباتتونو ميشه يه بار ديگه توضيح بديد
در ضمن كسي نمي خواد سوالي كه من گذاشتمو حل كنه؟
 

math1998

New Member
ارسال ها
336
لایک ها
224
امتیاز
0
پاسخ : ماراتن جبر (سطح پیشرفته)

ببخشيد من الان نه فهميدم شما چي رو مي خواين اثبات كنين نه نحوه ي اثباتتونو ميشه يه بار ديگه توضيح بديد
در ضمن كسي نمي خواد سوالي كه من گذاشتمو حل كنه؟
ببینید معمولا توابع فقط یک ضابطه دارن مگه تعداد خیلی اندکشون حالا شما وقتی به 2 یا بیشتر از یک جواب میرسین باید ثابت کنین که فقط یکی جوابه در واقع باید یکی از جواب ها رو در نظر بگیرید و ادامه بدید تا به تناقض برسید.
 

sina.a

New Member
ارسال ها
110
لایک ها
11
امتیاز
0
پاسخ : ماراتن جبر (سطح پیشرفته)

سوال:اعداد حقیقی
به گونه ای است که به ازای
,
است و
,بیشترین مقدار ممکن برای
را بدست آورید
 

golsefatan

New Member
ارسال ها
331
لایک ها
264
امتیاز
0
پاسخ : ماراتن جبر (سطح پیشرفته)

سوال:اعداد حقیقی
به گونه ای است که به ازای
,
است و
,بیشترین مقدار ممکن برای
را بدست آورید
راهنمایی: در نابرابری قدر مطلق تابع کمتر از 1 است مقادیر 0 و 0.5 و 1 را بگذارید و سعی کنید برای قدرمطلق های a,b,c روابطی بیابید.
 

aras2213

New Member
ارسال ها
216
لایک ها
228
امتیاز
0
پاسخ : ماراتن جبر (سطح پیشرفته)

سوال بعد:
دو چندجمله ای تکین و تجزیه ناپذیر در
هستند.اگرp,q دارای ریشه های
باشند که
نشان دهید چند جمله ای
یک ریشه گویا دارد.
این سوال، سوال واقعا قشنگیه(البته به نظر من) و کلا دید من رو نسبت به چندجمله ای عوض کرد.:25:
(من بعد از حل این سوال:4:)
 

golsefatan

New Member
ارسال ها
331
لایک ها
264
امتیاز
0
پاسخ : ماراتن جبر (سطح پیشرفته)

سوال بعد:
دو چندجمله ای تکین و تجزیه ناپذیر در
هستند.اگرp,q دارای ریشه های
باشند که
نشان دهید چند جمله ای
یک ریشه گویا دارد.
این سوال، سوال واقعا قشنگیه(البته به نظر من) و کلا دید من رو نسبت به چندجمله ای عوض کرد.:25:
(من بعد از حل این سوال:4:)
مگر سوال قبلي حل شد؟!!! اگر با اون راهنمايي حل شد لطفا اول راه حل و جواب را بگوييد بعد سوال بعدي رو بذاريد. ممنون از لطف شما...
 

Dadgarnia

New Member
ارسال ها
1,350
لایک ها
1,127
امتیاز
0
پاسخ : ماراتن جبر (سطح پیشرفته)

سوال بعد:
دو چندجمله ای تکین و تجزیه ناپذیر در
هستند.اگرp,q دارای ریشه های
باشند که
نشان دهید چند جمله ای
یک ریشه گویا دارد.
این سوال، سوال واقعا قشنگیه(البته به نظر من) و کلا دید من رو نسبت به چندجمله ای عوض کرد.:25:
(من بعد از حل این سوال:4:)
ما بايد تو اينگونه مسائل كه مشخص نكرده ضرايب چند جمله اي ها و ... عضو چه مجموعه اي هستند اونا رو مختلط در نظر بگيريم يا حقيقي؟
 

golsefatan

New Member
ارسال ها
331
لایک ها
264
امتیاز
0
پاسخ : ماراتن جبر (سطح پیشرفته)

ما بايد تو اينگونه مسائل كه مشخص نكرده ضرايب چند جمله اي ها و ... عضو چه مجموعه اي هستند اونا رو مختلط در نظر بگيريم يا حقيقي؟
از صورت سوال معلوم است که ضرایب چند جمله ای عضو اعداد گویا هستند. (q به معنای اعداد گویاست).
ولی دوستان اول سوال مربوط به بیشترین مقدار مجموع قددرمطلق ها رو حل کنید و بعد برید سراغ سوال بعدی.
اگر نیاز به راهنمایی بیشتر هست بگید تا راهنمایی کنم یا حل کنم. ممنون از لطف شما دوستان.
 

Dadgarnia

New Member
ارسال ها
1,350
لایک ها
1,127
امتیاز
0
پاسخ : ماراتن جبر (سطح پیشرفته)

جواب 17 ميشه.
اگه به جاي x بذاريم 0, 1, 0.5 بدست مياد:
پس بدست مياد:
و به همين ترتيب داريم​
حالا واضحه كه تابع
در شرايط مسئله صدق مي كنه.​
 
آخرین ویرایش توسط مدیر

math1998

New Member
ارسال ها
336
لایک ها
224
امتیاز
0
پاسخ : ماراتن جبر (سطح پیشرفته)

فرض f(n) تابعی جایگشتی باشد ان را بیابید.


[img/]






 

aras2213

New Member
ارسال ها
216
لایک ها
228
امتیاز
0
پاسخ : ماراتن جبر (سطح پیشرفته)

طبق نامساوی حسابی هندسی:

پس حالت تساوی رخ داده.به راحتی نتیجه میشه f(1)=1و تابع همانیه

---- دو نوشته به هم متصل شده است ----

سوال بعد:
دو چندجمله ای تکین و تجزیه ناپذیر در
(هم ضرایبشان گویاست و هم در مجمو عه ی اعداد گویا تجزیه ناپذیرند)هستند.اگرp,q دارای ریشه های
باشند که
نشان دهید چند جمله ای
یک ریشه گویا دارد.
 

math

New Member
ارسال ها
1,129
لایک ها
1,096
امتیاز
0
پاسخ : ماراتن جبر (سطح پیشرفته)

طبق نامساوی حسابی هندسی:

پس حالت تساوی رخ داده.به راحتی نتیجه میشه f(1)=1و تابع همانیه

---- دو نوشته به هم متصل شده است ----

سوال بعد:
دو چندجمله ای تکین و تجزیه ناپذیر در
(هم ضرایبشان گویاست و هم در مجمو عه ی اعداد گویا تجزیه ناپذیرند)هستند.اگرp,q دارای ریشه های
باشند که
نشان دهید چند جمله ای
یک ریشه گویا دارد.
سوال قبل با استفاده از نامساوی جایگشتی نیز نابود میشود !!!!

این سوال رو قبلا نگذاشته بودید ؟؟
 
بالا